• Matéria: Matemática
  • Autor: DanJR
  • Perguntado 8 anos atrás

Seja a equação \mathsf{p^n + 144 = q^2}, onde \mathsf{n} e \mathsf{q} são números inteiros positivos e \mathsf{p} é um número primo. Determine os possíveis valores de \mathsf{n}, \mathsf{p} e \mathsf{q}.


DanJR: É um desafio! Rs
Anônimo: que desafio tenso em
meurilly: verdade kkkkk
Anônimo: achei que p = 16 , n = 2 e q = 20
Anônimo: vamo continua olhando soluções
meurilly: Sim vamos.
GabrielMagal1: p tem que ser primo
GabrielMagal1: essa solução da pra perceber tambem , mas nao vale ne 16 nao é primo kk
Anônimo: ah verdade
Anônimo: não tinha lembrado

Respostas

respondido por: GabrielMagal1
4
 p^{n} +144 =  q^{2}

 p^{n} =  q^{2} -144

 p^{n} = (q+12).(q-12)

Para n≠2 , temos : 

 p^{n} =  p^{n-x} .  p^{x}

Como  p^{n} = (q+12)(q-12) , igualando os fatores : 

 p^{n-x} = q+12

 p^{x} = q-12

Então :

 p^{n-x} -  p^{x} = q+12-(q-12)

 p^{n-x} -  p^{x} = 24

Agora achamos primos que tem 24 como diferença entre 2 potencias , começando por p=2  : 

Teste para p=2 :

 2^{n-x} -  2^{x} = 24

 2^{5} -  2^{3} = 24  (potencias de 2 que satisfazem)

32 - 8 = 24 (V)

Logo para p = 2 , temos : 

x = 3

n-x = 5 ⇒ n = 3+5 ⇒ n=8

Calculando q : 

 2^{8} + 144 =  q^{2}

 q^{2} = 256+144

 q^{2} = 400  ⇒ q=20 

Nesse caso : p = 2 ; n = 8 ; q = 20.


Teste para p = 3 :

 3^{n-x} -  3^{x} = 24

 3^{3} -  3^{1} = 24 (potencias de 3 que satisfazem)

27 - 3 = 24 (V)

Logo , para p=3 temos :

x = 1 

n-x = 3 ⇒ n = 3+1 ⇒ n=4

Calculando q : 

 3^{4} +144 =  q^{2}

81+144 =  q^{2}

 q^{2} = 225

q = 15 .

Nesse caso : p = 3 ; n = 4 ; q = 15

*** É fácil perceber que para primos maiores que 3 não é possivel satisfazer a condição da diferença entre potencias igual a 24 . 


Voltando a  p^{n} =  q^{2} - 144 , para n = 2 :

 p^{2} =  q^{2} - 144

Podemos perceber outro caso , como q²-144 tem que ser positivo q tem que ser um numero a partir de 13 . 

Testando q =13 :

 p^{2} =  13^{2} - 144

 p^{2} = 169 - 144

 p^{2} = 25

p = 5 .

Nesse caso : p = 5 ; n = 2 ; q = 13

GabrielMagal1: Zero formalidade mas estão aí as soluções que encontrei kkk
DanJR: Olá Gabriel! Inicialmente, devo parabenizá-lo pela resolução. Também concluí isso!
DanJR: Mas,...
DanJR: [risos]
DanJR: Na parte final, especificamente em ***, afirmara não ser possível haver "p" primo maior que 3 cuja diferença de potências resulte em 24; mas achou p = 5.
GabrielMagal1: Valeu ;)
GabrielMagal1: Eu disse que o p não podia ser maior que 3 para o caso em que n≠2 , dá uma olhada
GabrielMagal1: Onde eu pus a restrição vou por que é para o caso de n≠2 para ficar melhor
GabrielMagal1: Não redigi bem essa parte , mas eu quis dizer que não valia para n≠2
DanJR: Ah! Sim!
Perguntas similares